Matematik

stationære punkt

27. oktober 2019 af Nanna34 - Niveau: B-niveau

nogen ka hjælpe mig  min hjerne ka ik fungere længere :///


Svar #1
27. oktober 2019 af Nanna34

#0

nogen ka hjælpe mig  min hjerne ka ik fungere længere :///


Brugbart svar (0)

Svar #2
27. oktober 2019 af peter lind


Brugbart svar (1)

Svar #3
27. oktober 2019 af peter lind

ey betragtes som en konstant når du differentierer med hensyn til x så fx(x,y) = 3*ey-3x2


Svar #4
27. oktober 2019 af Nanna34

#3

ey betragtes som en konstant når du differentierer med hensyn til x så fx(x,y) = 3*ey-3x2

hmm men er det ellers rigtigt iforhold til staionær punkt ?


Brugbart svar (1)

Svar #5
27. oktober 2019 af AMelev

For det første mangler du ' på den partielt afledede funktioner f_{x}{\color{Red} \mathbf{}'}(x,y)
Først skal du bestemme f_{x}{\color{Red} \mathbf{}'}(x,y) og f_{y}{\color{Red} \mathbf{}'}(x,y) - brenyt dit CAS-værktøj (så havde du opdaget, at dit første resultat var forkert).
For et stationært punkt gælder, at begge de partielt afledede er 0, så du skal løse de to ligninger f_{x}'(x,y)=0 og f_{y}'(x,y)=0 mht. x og y.  ?


Svar #6
27. oktober 2019 af Nanna34

#5

For det første mangler du ' på den partielt afledede funktioner f_{x}{\color{Red} \mathbf{}'}(x,y)
Først skal du bestemme f_{x}{\color{Red} \mathbf{}'}(x,y) og f_{y}{\color{Red} \mathbf{}'}(x,y) - brenyt dit CAS-værktøj (så havde du opdaget, at dit første resultat var forkert).
For et stationært punkt gælder, at begge de partielt afledede er 0, så du skal løse de to ligninger f_{x}'(x,y)=0 og f_{y}'(x,y)=0 mht. x og y.  ?

hvis det er rigtigt hvordan finder jeg så lokalt maksimum


Brugbart svar (0)

Svar #7
27. oktober 2019 af peter lind

I første omgang skal du finde det stationære punkter. Derefter skal du undersøge om det er et lokalt maksimum, lokalt minimum eller et saddelpunkt


Svar #8
27. oktober 2019 af Nanna34

#7

I første omgang skal du finde det stationære punkter. Derefter skal du undersøge om det er et lokalt maksimum, lokalt minimum eller et saddelpunkt

nårh et det noget med montoniforhold?


Brugbart svar (1)

Svar #9
27. oktober 2019 af mathon

                      \small \begin{array}{lllll} f_x=3e^y-3x^2&&&&f_y=3\cdot e^y\cdot x-3\cdot e^{3\cdot y}\\\\ f_{xx}=-6x&&&&f_{yy}=3\cdot e^y\cdot x-9\cdot e^{3\cdot y}\\\\ &&f_{xy}=3\cdot e^y \end{array}


Svar #10
27. oktober 2019 af Nanna34

#9

                      \small \begin{array}{lllll} f_x=3e^y-3x^2&&&&f_y=3\cdot e^y\cdot x-3\cdot e^{3\cdot y}\\\\ f_{xx}=-6x&&&&f_{yy}=3\cdot e^y\cdot x-9\cdot e^{3\cdot y}\\\\ &&f_{xy}=3\cdot e^y \end{array}

hmmm, men hva svarer fxx og fyy og fxy til?


Brugbart svar (0)

Svar #11
27. oktober 2019 af AMelev

Du må have noget materiale til dette emne - slå op i det. 


Skriv et svar til: stationære punkt

Du skal være logget ind, for at skrive et svar til dette spørgsmål. Klik her for at logge ind.
Har du ikke en bruger på Studieportalen.dk? Klik her for at oprette en bruger.